kimjy89
Thanks Received: 4
Vinny Gambini
Vinny Gambini
 
Posts: 24
Joined: May 17th, 2010
 
 
 

Q24 - Medical researcher: As expected, records

by kimjy89 Tue Jun 15, 2010 8:08 am

I understand why the correct answer would weaken the argument, but I am not quite sure why the other answers are wrong.

Thanks!
User avatar
 
ManhattanPrepLSAT1
Thanks Received: 1909
Atticus Finch
Atticus Finch
 
Posts: 2851
Joined: October 07th, 2009
 
This post thanked 1 time.
 
 

Re: Q24 - Medical researcher: As expected, records

by ManhattanPrepLSAT1 Tue Jun 15, 2010 2:48 pm

Fun and challenging question...

At first glance, this argument appears to be mistaking a correlation with a cause and effect relationship. However, the conclusion is that adequate prenatal care "reduces your risk" of low birth weight - that's different than saying that inadequate prenatal care causes low birth weight. The real problem must be in the statistics themselves. Otherwise, there wouldn't be much of an issue with the argument.

We are asked to find an answer choice that weakens the argument. Many of these are close, but turn out to be consistent with the argument.

(A) is tempting but consistent with the argument. The argument never claims that everyone with inadequate prenatal care would be born with low birth weight.
(B) undermines the argument by suggesting that how they're measuring the phenomena could be flawed, and that one characteristic could be construed as both having inadequate care and low birth weight. This would misconstrue the relationship and likelihood that those two phenomena overlap.
(C) relates being born premature and having a low birth rate. The conclusion is about prenatal care and birth weight, so misconstruing the relationship between these two factors would not undermine the conclusion.
(D) is consistent the argument's conclusion for the same reason that answer choice (A) is consistent - the argument never claims that no one with adequate prenatal care would be born with a low birth weight.
(E) is wrong for the same reason that answer choice (C) is wrong. It's relating premature birth with, adequate prenatal care, but premature birth is not related to the conclusion.

Clear things up? Let me know otherwise...
 
nflamel69
Thanks Received: 16
Atticus Finch
Atticus Finch
 
Posts: 162
Joined: February 07th, 2011
 
 
 

Re: Q24 - Medical researcher: As expected, records

by nflamel69 Sun Jan 27, 2013 8:12 pm

I thought this question deals with implicit causal reasoning..?
 
dl0120
Thanks Received: 1
Vinny Gambini
Vinny Gambini
 
Posts: 4
Joined: November 08th, 2012
 
 
 

Re: Q24 - Medical researcher: As expected, records

by dl0120 Wed Jan 30, 2013 9:50 pm

I am looking at B and E. I've read Kaplan's, Graeme Blake's, and Manhattan's explanations, but none of them addressed the following things:

B: B would weaken the argument if it ended the sentence at prenatal care. But it adds the following: "when the record of that care is not available." They don't ALWAYS routinely classify mothers giving premature birth as having received inadequate prenatal care. They do it "when the record of that care is not available." This brings up some questions:

1. How frequent is this? Maybe it happens 1/1000 births, and maybe it happens 8/10 births. If it's the former, it wouldn't matter so much because it'd be just a "some" statement - some mothers giving premature birth are classified as having received inadequate prenatal care. This wouldn't weaken the argument, the same way D doesn't impact the argument. Just because there are some, doesn't mean the reasoning is weakened. After all, the conclusion is "significantly decreases," not "completely prevents."

If it happens 8/10 births, then it does weaken the argument, because a significant proportion of the data is unreliable. Unfortunately, the choice just says "when the record is not available," giving us no suggestion of whether we should take it as "some" or "significantly enough."

2. Just as importantly or more so, this classification happens when the record of that care is not available. Considering that the stimulus is talking about "ten major hospitals," it's NOT INCONCEIVABLE to think that when the RECORD of care isn't available, the care was probably not adequate. It's really common sense that hospitals are strict with their records, and keeping a record of patient data is a big deal. I think an average American with common sense MAY assume that the fact that a major hospital could NOT find the records for a patient is NOT a strong indication of that patient's receiving adequate prenatal care.

So I thought this: that I have NO idea if this routine classification happens "some" times or "significantly enough" number of times. I starred it and moved on to E.

E says women with adequate prenatal care are less likely to have premature births. This says that adequate prenatal care may NOT directly be related to less risk of low body weight babies. This places the chance of premature birth BETWEEN adequate prenatal care and risk of low birth weight babies - just like an alternative cause.

I thought about whether I should think of it as a METHOD of impact or an ALTERNATIVE cause. If lessening the risk of premature birth is considered as a way by which adequate prenatal care CAUSES the decreased risk of low birth weight babies, E strengthens the argument.

If lessening the risk of premature birth is considered as an ALTERNATIVE cause, then E weakens the argument - it's NOT adequate prenatal care that directly causes the risk of low body weight babies to go down. It's the act of premature birth.

Typically on the LSAT, the Cause causes the Effect, nothing else. For example, on Manhattan book page 115 (PT28, S3, Q16), when the stimulus says "venereal disease caused his deafness," it is saying that "venereal disease DIRECTLY caused deafness." It's NOT saying "venereal disease caused Beethoven to ingest mercury. Mercury caused deafness. Therefore venereal disease caused the deafness."

Returning to this question, I thought it was POSSIBLE for E to weaken the argument by showing "premature birth" as the direct cause, and adequate prenatal care makes it less likely to experience "premature birth." I thought it was NOT a clear right answer or a wrong answer.

So I basically see B and E as two unclear answers. If I encounter this uncertainty in a real test, I'm not sure how I'd be able to choose the right answer with certainty. Please help me differentiate between the two answers in these respects!
 
monygg85
Thanks Received: 1
Jackie Chiles
Jackie Chiles
 
Posts: 29
Joined: December 04th, 2012
 
 
 

Re: Q24 - Medical researcher: As expected, records

by monygg85 Wed Sep 04, 2013 8:00 pm

This was one tough mother!

Sorry couldnt resist! Ha :)

Anyway, I got this right by narrowing it down to B and C and "sort of" guessing. Upon review it dawned on me that it was pretty tough and I got away with the whole question. Upon review I was confused between B C and E. I could give reasons to eliminate C and E...but I didnt think they were very.. valid. As for B, I had a hard time understanding why its the answer.

So here goes an explanation..or two:

Core: Records from 4 years covering 10 hospitals indicate that prematurely born babies were more likely to have low birth weight than not prematurely born. These records also state that mothers receiving adequate prenatal care were less likely to have low birth weight babies than parents receiving inadequate.

Conclusion: Adequate prenatal care significantly reduces the risk of having a low birth baby.

(A) Doesnt weaken. The conclusion can still stand. More or less we are concerned with low/not low birth weight babies, not normal.

(B) I had a hard time with this one. I guess its connecting that premature (low weight babies) are routinely classified as getting inadequate care when records arent available. This weakens (even if a tiny bit!) by suggesting that the connection between premature babies and inadequate prenatal care isnt so black and white, even more so, that maybe adequate care wouldnt significantly reduce low birth...it still may just a LITTLE but not significantly...maybe its the baby being born premature is to blame since thats purposefully being lumped in with inadequate/adequate care (according to this answer choice)

(C) It was a bit tough for me but I think this answer could potentially strengthen (if theres a connection between premature born and inadequate care) the argument or just not do anything, since it connects prematurely born babies with low birth weight and thats not what we need. The conclusion is based on adequate care and low birth.

(D) That wouldnt weaken it bc the conclusion allows for "some"

(E) I felt like, if anything, this would strengthen the fact that adequate prenatal care reduces risk of low birth because adequate care = less likely to be premature = less likely to have low birth. Thats assuming the connection does exist between these variables. If it not this, then at least I knew this doesnt weaken the conclusion.


I would appreciate any help on this! For me it was one of the harder questions I have encountered
 
rpcuhk
Thanks Received: 5
Forum Guests
 
Posts: 41
Joined: May 02nd, 2011
 
 
 

Re: Q24 - Medical researcher: As expected, records

by rpcuhk Mon Jun 16, 2014 9:19 pm

To Matt,

I don't think (E) is wrong because, as you said "premature birth is not related to the conclusion." Prematurely born babies are more likely to have low birth weights. So if as (E) said, adequate prenatal care decrease the likelihood of premature birth, then it can be inferred that adequate prenatal care decreases the risk of low birth weight babies, which actually strengthen the argument.
 
jrnlsn.nelson
Thanks Received: 2
Vinny Gambini
Vinny Gambini
 
Posts: 24
Joined: September 06th, 2014
 
 
 

Re: Q24 - Medical researcher: As expected, records

by jrnlsn.nelson Fri Oct 17, 2014 11:24 am

Gambini --

Great post, I see where you're coming from. I wish I could provide some help, but I can't. I agree that E is more right than B!

If any vets out there can explain this I'll be impressed. Would be interesting to know Graeme's specific thoughts.
 
pewals13
Thanks Received: 15
Elle Woods
Elle Woods
 
Posts: 85
Joined: May 25th, 2013
 
 
 

Re: Q24 - Medical researcher: As expected, records

by pewals13 Tue Nov 18, 2014 3:40 pm

I think the key to eliminating (E) is to stay focused on the precise wording of the conclusion.

If adequate prenatal care reduces the chance of premature birth and premature birth increases the risk of low birth weight babies, it is still true that adequate prenatal care makes low weight babies less likely.
User avatar
 
WaltGrace1983
Thanks Received: 207
Atticus Finch
Atticus Finch
 
Posts: 837
Joined: March 30th, 2013
 
 
trophy
Most Thanked
trophy
Most Thankful
trophy
First Responder
 

Re: Q24 - Medical researcher: As expected, records

by WaltGrace1983 Tue Jan 27, 2015 3:03 pm

Records: premature babies, more likely to have low birth weight
+
Records: adequate prenatal care, less likely to have low birth weight
-->
Adequate prenatal care significantly decreases risk of low birth weight

It is very important to say, as mentioned above, that the conclusion is NOT "adequate prenatal care causes low birth weight," or something like that. We are talking about decreasing risk.

(A) Some babies have inadequate prenatal care and normal birth weight. This is consistent with the argument and so doesn't help/hurt it. The argument is not saying that ALL babies with inadequate care all have low birth weights. It simply says that they are "less likely."

(C) Premise booster. This is saying that many premature babies were recorded as having a low birth weight. We already know this.

(D) This is the same as (A), just with a different group.

(E) Strengthens. If adequate prenatal care is less likely to give birth prematurely, and less likely to give birth prematurely would result in less likely to have low birth weight, then the conclusion that (adequate prenatal care --> decreases risk of low birth weight) would be strengthened.

This gives us (B). (B) essentially says that the basis of this argument, the records, might not be representative of reality. This is hugely significant as it would show that adequate prenatal care might NOT have much of an effect on the risk of low birth weight.

Also, I think that this stem is worded very deliberately. Whenever I see "most weakens" as opposed to "weakens," I think the answers are a little bit softer. I could be wrong, but it is just something that I have noticed.
 
deddiekated
Thanks Received: 0
Vinny Gambini
Vinny Gambini
 
Posts: 6
Joined: September 21st, 2016
 
 
 

Re: Q24 - Medical researcher: As expected, records

by deddiekated Wed Apr 19, 2017 5:47 am

Not Sure if anybody still reads these blogs, but if anybody does I'd appreciate any input on how I approached this problem:

Premise 1: Prematurely born babies are more likely to have low birth weights.
Premise 2: Mothers who received adequate prenatal care are less likely to have low birth weights.

conclusion: Adequate prenatal care decreases the risk of low birth weight babies.

I thought the hidden assumption was that Adequate Prenatal care decreases the likelihood of having prematurely born babies.

So it was rather easy to eliminate all other choices, as well as (E) which pretty much is the assumption / strengthens the argument.
Whereas, (B) weakens the argument by saying that the assumption of the argument is flawed...

Any input would be nice! Thank you!
User avatar
 
ohthatpatrick
Thanks Received: 3808
Atticus Finch
Atticus Finch
 
Posts: 4661
Joined: April 01st, 2011
 
This post thanked 2 times.
 
 

Re: Q24 - Medical researcher: As expected, records

by ohthatpatrick Wed Apr 19, 2017 8:58 pm

Once you've read an argument and found the conclusion, I recommend doing the following things.

[b]1. Say the Conclusion, and then ask "how come? how do we know? what's the support?"
Adequate prenatal makes you less at risk of low birth wt babies.
how do we know?

[b]2. Find the Supporting reason(s).
Because there's a correlation between adequate prenatal care and low birth weight.

Does the correlation between born prematurely and low birth weight matter?

Not yet. We might be anticipating that they're inserting this filler for the sake of using it later or just for the sake of making more concepts available for trap answers.

But given that 1st sentence and 2nd sentence are each a correlation, we obviously care more about the 2nd one, since THAT correlation connects the cause/effect in the author's conclusion.

We should just prephrase this like we do ALL causal conclusions:
1. Is there some OTHER WAY to explain the Evidence?
2. How PLAUSIBLE is the Conclusion?

In the terms of this argument,
1. Is there some OTHER WAY to explain why there's a correlation between adequate prenatal and low birth weight?

or

2. What would decrease the plausibility that prenatal care has an effect on the birth weight of the baby?

(B) is correct because it performs the function of #1.

It suggests that the reason prenatal and low birth weight are correlated is NOT because prenatal actually causally affects birth weight, but rather because being born premature leads to both low birth weight and to being classified as "inadequate prenatal care".

=== analogy ===
As expected, people who make more than $200k/yr are more likely to drive a luxury car than those who don't. But we have also found that people who eat caviar are more likely to drive a luxury car than those who don't. Eating caviar, therefore, significantly increases your desire to drive a luxury car.

(B) would be saying, "Eating caviar is NOT having a causal influence on your owning a luxury car. There's a correlation between caviar and luxury cars because they are BOTH SYMPTOMS of the same underlying CAUSE: making more than $200k/yr.
 
WhimsicalWillow
Thanks Received: 0
Vinny Gambini
Vinny Gambini
 
Posts: 12
Joined: February 07th, 2020
 
 
 

Re: Q24 - Medical researcher: As expected, records

by WhimsicalWillow Fri May 29, 2020 12:08 am

Classic Phenomena Hypothesis Weakening Question with Correlation-Causation Flaw.


Conclusion: Adequate prenatal care decreases the risk of low birth weight babies

Why?
Premise:
1. Records show premature babies tend to have lower birth weights
2. Records show adequate prenatal care makes it LESS LIKELY they'll have low birth weight babies

We must weaken by asking:

1) Can an alternative explanation (hypothesis) explain this (the phenomena)?
2) Does something weaken the truth/dents the credibility of the data in the explanation(hypothesis)?


We must weaken by taking a skeptical stance. Think like a debater:
It's NOT that A (Adequate prenatal care) decreases the risk of B (low birth weight) babies)... it is because of C...

A & D give the same logical forms of trap ACs. They both are consistent with the argument yet try to trick you into thinking they contradict premise. Both A&D give possibilities that are already included in the argument due to it being a correlation which allows for people/data who don't conform.

A) No, consistent with argument
“The records indicate that babies born with normal birth weights had mothers who had inadequate prenatal care”
/low birth wt-->/adequate prenatal care

D uses opposite groups and gives the same logical challenge as A

D) No, consistent with argument
“Some babies not born prematurely had low birth weights and mothers who received adequate prenatal care”
low birth weights--> adequate prenatal care

A&D do not weaken because these possibilities are already included in the correlation in the stimulus because correlations allow for people that don't conform/nonconforming data points. It's not a conditional statement but a correlation.

Our groups are x) low birth weight babies and y) adequate prenatal care
C & E are both incorrect because of the irrelevant term shift of the groups: z) premature, which is irrelevant to the conclusion. We don't care about it.

C brings up irrelevant term x) low birth weight babies and z) premature

E brings up irrelevant term y) adequate prenatal care and z) premature

B) Correct because it dents credibility of the data which addresses question #2 "2) Does something weaken the truth/dents the credibility of the data in the explanation(hypothesis)?" . If it's the hospital, by default, classifies them having "inadequate prenatal care" when they actually don't have record of the care at all, we cannot trust the data. It weakens the records used to justify the phenomena in the hypothesis.